Write 5.4 as a mixed number and as an improper fraction.
Write your answers in simplest form.

Answers

Answer 1

Answer:

5.4 as a fraction is 27/5.

Step-by-step explanation:

Learned in 5th grade


Related Questions

if candy canes cost .89 a dozen, how much would it cost to buy candy canes for a school with 300 students

Answers

Answer:

267

Step-by-step explanation:

Answer:

First dont give candie second keep for yourself third mock them 4th be greedy

Step-by-step explanation:

by buying candie from your own money nah waste on yourself

An ice cream store has 31 flavors of ice cream and 10 toppings. A regular sundae has one flavor of ice cream, one topping, and comes with or without whipped cream. How many different ice cream sundaes can be ordered?

Answers

10 because there is only 10 toppings and each only comes with one topping!

Answer:

610

Step-by-step explanation:

31 x 10= different flavour and topping combination

x2 = for the whipped cream different option

M.) A typical tip in a restaurant is 15% of the
total bill. If the bill is $125, what would the
typical tip be?

Answers

The tip should be 18.75

Which of the following statements is meaningless?
A.There was a 3% decrease in the price of sugar.
B.There was a 200% increase in the population of a town.
C.There was a 120% decrease in the amount of water in a dam. D.There was a 0% change in the temperature in the room after 1 hour.

Answers

Answer:

(C)

Step-by-step explanation:

If can't have a meaning as something can't decrease more than 100% of itself.

The statement that is meaningless is

There was a 120% decrease in the amount of water in a dam.

Option C is the correct answer.

What is a percentage?

The percentage is calculated by dividing the required value by the total value and multiplying by 100.

Example:

Required percentage value = a

total value = b

Percentage = a/b x 100

Example:

50% = 50/100 = 1/2

25% = 25/100 = 1/4

20% = 20/100 = 1/5

10% = 10/100 = 1/10

We have,

A percentage increase can be of any number.

A percentage decrease can not be below 100%.

Now,

A.

There was a 3% decrease in the price of sugar.

This is possible.

B.

There was a 200% increase in the population of the town.

This is possible.

C.

There was a 120% decrease in the amount of water in a dam.

This is not possible.

The highest percentage decrease can be up to 100%.

D.

There was a 0% change in the temperature in the room after 1 hour.

This is possible.

Thus,

There was a 120% decrease in the amount of water in a dam is meaningless.

Learn more about percentages here:

https://brainly.com/question/11403063

#SPJ2

Miguel bought 8 equally priced pens for $3.84.

How much will 15 of these pens cost?

Answers

Answer:

$7.20

Step-by-step explanation:

If 8 pens cost $3.84, we need to find out how much ojne pen costs.

To find out the cost of one pen we need to divide $3.84 by 8.

$3.84 / 8 = $0.48

1 pen cost $0.48

We then need to work out the cost of 15 pens. So from using the information that one pen costs $0.48 we can multiply that by 15 to find out the answer.

$0.48 x 15 = $7.20

15 pens cost $7.20

Lizzy wants to solve the equation 6(2x + 1) = 4x – 5. Which step would be an appropriate first step for Lizzy to take to solve for x?

Answers

Answer:

distribute parenthesis

Step-by-step explanation:

6(2x + 1) = 4x - 5 ← distribute parenthesis on left side

12x + 6 = 4x - 5 ( subtract 4x from both sides )

8x + 6 = - 5 ( subtract 6 from both sides )

8x = - 11 ( divide both sides by 8 )

x = - [tex]\frac{11}{8}[/tex]

Maggie has a collection of 1,200 Pennies. Of these, 25% are dated before 1980, 35% are dated from 1980-2000.and the rest are dated after 2000
How many pennies in Maggie's collection are dated after 2000?
A 480 B 720 C 40 D 60

Answers

Answer:

A- 480

Step-by-step explanation:

First, I'm going to work out 25% of 1200 for the before 1980 pennies.

10% = 120

20% = 240

5% = 60

240 + 60 = 300 pennies before 1980.

Next, work out 35% of 1200 instead.

20% = 240

+ 10%

30% = 360

+ 5% = 60

35% = 420 pennies from 1980-2000

Finally, work out how many pennies are leftover.

300 + 420 = 720

1200 - 720 = 480

A is the answer.

Hope this helps and have a good day!

PLS HELP SO CONFUSED POSSIBLE BRANLIEST

The figure below shows the letter Z and four of its transformed images—A, B, C, and D:

A coordinate plane is shown. The letter Z has endpoints at negative 6 comma 3, negative 6 comma 1, negative 4 comma 1 and negative 4 comma 3 and has a diagonal line that goes from top right to bottom left in between. A is a shape that has endpoints at 4 comma 3, 4 comma 1, 6 comma 3 and 6 comma 1 and has a diagonal line that goes from top left to bottom right. B has endpoints at negative 6 comma negative 1, negative 6 comma negative 3, negative 4 comma negative 1 and negative 4 comma negative 3 and has a diagonal line that goes from top right to bottom left. C has endpoints at negative 3 comma negative 1, negative 3 comma negative 3, negative 1 comma negative 1, negative 1 comma negative 3, and has endpoints that go from top left to bottom right. D has endpoints at 2 comma negative 1, 2 comma negative 3, 4 comma negative 1, and 4 comma negative 3 and has a diagonal that goes from top right to bottom left.

Which of the four images was formed by a reflection of the letter Z?

A
B
C
D

Answers

Answer: The correct answer is A.

Step-by-step explanation: The answer is A, because a reflection would be reflected, therefore it is on the right side of the graph and the Z is facing the other way as if it were a literal reflection.

A

give the other brainliest

PLEASE HELP ME IT'S DUE TODAY AND IT COULD HELP MY GRADE A LOT

Answers

angle is 90 degrees and i believe hypotenuse is 140

Answer:

Hypotenuse: 20√2

Acute Angle measurements: 45

Step-by-step explanation:

Note that the two leg sides are both 20 measurements, suggesting that it is an isosceles triangle.

By definition of a isosceles triangle, the angles will be 45-45-90, and the sides will follow the 1-1-√2 rule, in which the 1 stands for the legs, and the √2 stands for the hypotenuse.

Note that the leg side measures 20. Multiply:

20 * √2 = 20√2.

Hypotenuse: 20√2

Acute Angle measurements: 45

what id the limit of this function? f(x)=2x4+5x3−2x+12

Answers

Your math problem

(==)=24=+53−2+12

f(x)=2x^{4}+5x^{3}-2x+12

Find solutions on the

cannot decide which of two washing machines to buy. The selling price of each is ​$. The first is marked down by ​%. The second is marked down by ​% with an additional ​% off. Find the sale price of each washing machine. Use pencil and paper. Explain why should buy the first washing machine rather than the second if the machines are the same except for the selling price.

Answers

You should do 585 × 0.50 to find the markdown. Then subtract the markdown from the selling price to find the sale price.

Step-by-step explanation:

Which geometric series converges? StartFraction 1 Over 81 EndFraction StartFraction 1 Over 27 EndFraction one-ninth one-third ellipsis 1 one-half one-fourth one-eighth ellipsis Sigma-Summation Underscript n = 1 Overscript infinity EndScripts 7 (negative 4) Superscript n minus 1 Sigma-Summation Underscript n = 1 Overscript infinity EndScripts one-fifth (2) Superscript n minus 1.

Answers

The geometric series that converges from the listed option is [tex]1+\frac{1}{2} + \frac{1}{4} +\frac{1}{8} ,...[/tex] Option B is correct.

The nth term of a geometric sequence is expressed as;

[tex]a_n=ar^{n-1}[/tex]

a is the first term of the sequencer is the common ration is the number of terms.

For a geometric sequence to converge, the modulus of its common ratio must be less than 1 (|r| < 1), otherwise, it diverges.

For the first series given as [tex]\frac{1}{81} + \frac{1}{27} +\frac{1}{9} +\frac{1}{3}+ ,...[/tex]

[tex]r = \frac{1}{27} \div \frac{1}{81}\\r=\frac{1}{27} \times 81\\r=\frac{81}{27} \\r=3 > 1[/tex]

Since the common ratio of the sequence is greater than 1, hence the series diverges.

For the series [tex]1+\frac{1}{2} + \frac{1}{4} +\frac{1}{8} ,...[/tex]

[tex]r=\frac{1/2}{1} =\frac{1/4}{1/2} = \frac{1}{2}[/tex]

Since the common ratio of the sequence is less than 1, hence the series converges.

Learn more on convergence of series here: https://brainly.com/question/14294471

A bakery made 24 cherry pies, using 114 cherries for each pie. They threw away 35 cherries that were bad. If they used all the cherries they had, how many cherries did they start off with?

Answers

Answer:

2,771

Step-by-step explanation:

24 x 114 = 2736 + the 35 they threw away = 2,771

Twice the difference of a number and 2 is at least 16 use the variable y for the unknown number

Answers

Answer:

2(y-2) is greater then or equal to 16

Step-by-step explanation:

f(x) is graphed below. If g(x)=f(x+2)-4, what are the roots of g(x)?

Answers

Answer:

fortnit 90s

Step-by-step explanation:

crank and c until u get too 90 height

I seriously don’t understand please help

Answers

Answer:

Me two I'm dum lol I think it 2× 9=18 in to 0 go 0 times

A boat heading out to sea starts out at Point AA, at a horizontal distance of 968 feet from a lighthouse/the shore. From that point, the boat’s crew measures the angle of elevation to the lighthouse’s beacon-light from that point to be 7^{\circ} ∘ . At some later time, the crew measures the angle of elevation from point BB to be 3^{\circ} ∘ . Find the distance from point AA to point BB. Round your answer to the nearest tenth of a foot if necessary.

Answers

As the distance from the lighthouse, LH, increases, the angle of elevation,

reduces.

The distance from point A to point B is approximately 1,269.9 feet.

Reasons:

The distance of point A from the light house = 968 feet

Angle of elevation from point A to the lighthouse =

Angle of elevation from point B to the lighthouse =

Required:

The distance from point A to point B.

Solution:

[tex]\displaystyle tan(\theta) = \frac{Opposite \, side \, to\, angle}{Adjacent\, side \, to\, angle} = \mathbf{\frac{Height \ from \ sea \ to \ the \ light \ house }{Horizontal \ from \ boat \ to \ the\ LH}}[/tex]

Which gives;

[tex]\displaystyle tan(7^{\circ}) = \mathbf{\frac{Height \ from \ sea \ to \ the \ light \ house }{968 \ feet}}[/tex]

Height to the lighthouse = tan(7°) × 968 feet  ≈ 118.86 feet

From point B, we have;

[tex]\displaystyle tan(3^{\circ}) = \mathbf{ \frac{118.86 \ feet }{Horizontal \ distance \ from \ point \ B\ to \ LH}}[/tex]

Therefore;

[tex]\displaystyle Horizontal \ distance \ from \ boat \ to \ LH = \frac{118.86 \ feet }{tan(3^{\circ})} \approx \mathbf{2237.9 \, feet}[/tex]

Therefore;

Distance from A to B = D

D = Horizontal distance from B to LH - Horizontal distance from A to LH

Distance from A to B = 2237.9 feet - 968 feet ≈ 1269.9 feet

The distance from point A to point B, D ≈ 1,269.9 feet

Learn more about trigonometric ratios here:

https://brainly.com/question/16577559

https://brainly.com/question/9107775

What is the solution to the system of equations?

Answers

Answer:

(-2, 5/3)

Step-by-step explanation:

Substitute x with -2 and solve

y = 2/3(-2) + 3

Multiply 2/3 and -2

y = -4/3 + 3

Add

-4/3 + 3 = 5/3

y = 5/3

Final answer:

(-2, 5/3)

0.36 as a standard form

Answers

Answer:

0.36 x 10⁰

Step-by-step explanation:

Find the unit rate.

289 miles on 10 gallons

The unit rate is( )miles per gallon

Answers

Answer:

28.9 mpg

Step-by-step explanation:

Divide the miles by the gallons. 289 / 10 = 28.9

113 + 377 = 117 + _

Pls answer guys.

Answers

Answer:

373

Step-by-step explanation:

113+377=490

490-117=373

answer:

=117+373

The table below shows the price of rolls at a bakery. A baker’s dozen includes 13 rolls for the price of a dozen rolls. How much money is saved by buying a baker’s dozen instead of 13 individual rolls?

Number of rolls
6
8
Baker’s dozen
Price ($)
$3.60
$4.80
$7.20
$0.55
$0.60
$0.70
$0.75

Answers

Answer:

0.60 cents

Step-by-step explanation:

Answer: 0.60

Step-by-step explanation:

Twelve is sixteen less than four times a number. Write it as a mathematical statement.

Answers

Answer:

12=4x-16

x would represent the unknown number.

Which ordered pair is a solution of the equation?
y + x = 3(x – 4)

Answers

x=1/2 y+6 . Hope it helps.

Write the equation of the line that passes through the point -3,4 and slope of 6

Answers

Answer: y=6x+22
Explanation:
1. we first start with the equation y=Mx+b
2. Then we plug in the slope (6) into M
3. Now you have y=6x+b
4. Now you plug in your point (-3,4) into the y and the x values
5. You are now stuck with 4=6(-3)+b
6. Now multiply 6 and -3 which gets you -18
7. Your new equation is now 4=-18+b
8. Now add 18 to both sides of the equals sign
9. Now you have 4+18=b
10. Add 4+18 which gets you 22. B is 22
11. Plug b into the equation and turn the values of x and y back to those letters
12. You now have the answer of y=6x+22

Two concentric circles are of radii 5 cm and 3 cm. Determine the length of the chord of the larger circle which touches the smaller circle.​

Answers

Answer:

Let O be the common center of two concentric circles and let AB be a chord of larger circle touching the smaller circle at P join OP

Since OP is the radius of the smaller circle to any chrod of the circle bisects the chord.

∴ AP=BP

In right ΔAPO we have

OA  

2

=AP  

2

+OP  

2

⇒25−9=AP  

2

 

⇒AP  

2

=16⇒AP=4

Now AB=2,AP=2×4=8[∵AP=PB]

hence the length of the chord of the larger circle which touches the smaller circle is 8cm.

Step-by-step explanation:

A set of 6 art books costs $241.25. A copy of one of the books, bought separately, costs $38.25. About how much less is the cost of the 6 books if you buy the set?

Answers

The cost of the 6 books if you buy the set is $11.75 less.

The given parameters:

Total number of books, n = 6Cost of the books, c = $241.25Cost of one book, = $38.25

The total cost of the 6 books when bought separately is calculated as follows;

Cost = 6 x $38.25

Cost = $229.5

The difference in price is calculated as follows;

= $241.25 - $229.5

= $11.75

Thus, the cost of the 6 books if you buy the set is $11.75 less.

Learn more about cost price here: https://brainly.com/question/19104371

Sam has a piece of rope that is 3. feet long. He cuts the rope so that one piece
is 13 feet long. How long is the other piece?
2 feet
2 feet
27 feet
12 feet
Done

Answers

Answer:

1.11 feet

Step-by-step explanation:

i assume you meant inches not feet so you will just multiply 3 feet by 12 to change it into 36 inches do 36-13 get 33 dived by 12 get 1 with a remainder of 11 so 1.11

1. Calculate the mean, median, and mode of the following set of data. Round to the nearest tenth.
7,3, 2, 1, 13, 8, 1, 5, 14, 11, 15
ANSWERS
A.
mean=6.7, median =7,mode=8
B.
mean=7.3, median =8,mode=1
C.
mean=7.3, median =7,mode=1
D.
mean=6.7,median =7,mode=1

Answers

Answer:

C.

Step-by-step explanation:

7,3, 2, 1, 13, 8, 1, 5, 14, 11, 15

Write them in ascending order:

1, 1, 2, 3, 5, 7, 8, 11, 13, 14, 15.

The mean = sum of the numbers / 11

= 80/77

= 7.3.

The median = the middle number

= 7.

The mode = the number which occurs most

= 1.

Which equation is equivalent to the formula r = st? t equals s over r t = rs s equals r over t s = rt.

Answers

Answer:

Step-by-step explanation:

r = st------------

Other correct expressions:

t = r/s

s = r/t

------------------

Choices

1)  t = s/r   No

2) t = rs     No

3)  s =r/t    Yes

4)  s = rt    No

The equation that is equivalent to the given equation is [tex]s = r/t[/tex] and this can be determined by using the arithmetic operations.

Given :

Equation is [tex](r = st)[/tex].

The following steps can be used in order to determine the equation that is equivalent to the given equation:

Step 1 - Write the given equation.

[tex]r = st[/tex]    --- (1)

Step 2 - The arithmetic operations can be used in order to determine the equation that is equivalent to the given equation.

Step 3 - Now, divide 't' on both sides in the equation (1).

[tex]\dfrac{r}{t} = \dfrac{st}{t}\\\\\\dfrac{r}{t}=s[/tex]  

Therefore, the correct option is C).

For more information, refer to the link given below:

https://brainly.com/question/11897796

Other Questions
help please i wonder???????????? Capstone Project part 11 quizWhat is the following code snippet checking? (1 point)while response != "yes":Othat response variable is not yet yesO that response variable is yesO if "yes" is indexed in an array for the variable "response"O if the response variable is "no"If you give all answers you will have an amazing Christmas! Summarize: Complete the following statements:A. 1 mole of any element contains 6.0221 x 1023 _____B. 1 mole of any compound contains 6.0221 x 1023 ______ please answer the geography questions im so confused and i need to know this for my exam. which one of the following items would never appear on a cash budget? group of answer choices office salaries expense interest expense depreciation expense travel expense 34z+6= i dont know how to simplify it In MNO, mM=(6x+1)mM=(6x+1) , mN=(3x10)mN=(3x10) , and mO=(x+19)mO=(x+19) . Find mN.mN. A scientist is working with 0.9 of gold wire. How long is the wire in millimeters? Which ones colder? -5 degrees Celsius or -10 degrees Celsius What happens at the end of loser by jerry spinelli The speaker of which statement would be best served by joining an interest group?. What is your favourite subject and why? y = 6x - 4 when x = -7 The graph of part of linear function g is shown on the grid.Which inequality best represents the domain of the part shown? SOMEBODY HELP NO LINKS PLZ AND THANK YOUgiven the table below which function shos an exponential function table x f(x) g(x) h(x)1 64 65 5 2 32 54 7 3 16 43 10 4 8 32 14 A f(x)B g(x)C h (x)D none of the above Find an equation of the plane.The plane that contains the line x = 3 + 2t,y = t,z = 6 t and is parallel to the plane 2x + 4y + 8z = 16 Determine if the data below represents a function. Explain your reasoning. (3, 2), (4,7), (8, 12). (10, 15) How do I simplify the ratio of 42 : 28 : 21 FOR 20 POINTS r(x)=7x3 and q(x)=r(3x).What equation shows the correct rule for the function q?q(x)=21xq(x)=21x3q(x)=21x+9q(x)=21x+3 Find the value of y 7=6+y